IMO олимпиада по математике 1994 года | Казахстанские олимпиады

Пусть mm и nn — целые положительные числа. Пусть a1{{a}_{1}}, a2{{a}_{2}}, \ldots , am{{a}_{m}} —различные элементы множества {1,2,,n}\left\{ 1,2,\ldots ,n \right\} такие, что для любых индексов i,ji,j, удовлетворяющих условиям 1ijm1\le i\le j\le m и ai+ajn{{a}_{i}}+{{a}_{j}}\le n, существует индекс kk, 1km1\le k\le m, для которого ai+aj=ak{{a}_{i}}+{{a}_{j}}={{a}_{k}}. Доказать, что a1+a2++ammn+12\dfrac{{{a}_{1}}+{{a}_{2}}+\ldots +{{a}_{m}}}{m}\ge \dfrac{n+1}{2}.

Решение

Здесь могут быть решения задач с LaTeX\LaTeX